Which one of the following, if substituted for the condition that motorbike servicing has to be earlier than laundry,...

Valerie on November 17, 2016

How to approach

Hello, I'm wondering if there is some strategy on how to approach this type of questions, where the substitutions are the answer choices. While it isn't a difficult question, It takes much time to try out each of the answer choices. Thank you.

Replies
Create a free account to read and take part in forum discussions.

Already have an account? log in

Mehran on December 2, 2016

@Valerie unfortunately, not really.

The key for these questions is your initial setup and all the deductions you were able to make, because the correct answer choice will result in the exact same thing when substituted for the condition referenced in the question stem.

Any difference will eliminate the answer choice as a possibility.

Hope that helps! Please let us know if you have any other questions.

Miller on November 2, 2017

Can you please help me get ne'er stand why Zander choice (c) is incorrect when J earlier than L would have the same effect as M earlier than L if M and J are connected?

Christy-Earls on May 10, 2019

Would love video setup for this game...

Ravi on May 10, 2019

@Miller and @Christy-Earls,

Happy to help.

@Miller, Could you clarify what you're referring to? (C) is actually
the correct answer choice for this problem; it isn't incorrect.

Regarding this question, we know that in the original rules, M had to
go before L. We also know that M has to be either immediately before
or immediately after J. Combining these two rules, we had

JM - L or MJ - L

The question tells us to pick a rule that, if substituted for the rule
that M has to go before L, would have the same effect.

In looking at the original rule, what do we notice? Well, in addition
to M going before L, J must also go before L since it has to come
either right before or right after M. Since the M before L rule is
gone, if we substituted a J before L rule, then we'd create the same
effect in determining the order of the student's activities.

This leads us to (C), which says J has to go before L. Because the
last rule of the game still exists (M has to go either right before or
right after J), this rule substitution creates the same scenarios.

JM - L or MJ - L

Thus, (C) is the correct answer.

@Christy-Earls,

If you have specific video requests for games, please direct any
support related issues to our support staff by tapping "support" from
the left menu or by calling 855.483.7862 ext. 2 Monday-Friday 9am-6pm
PT.

In the meantime, let us know if you have any questions, and we'll be
happy to provide written explanations for any LSAT questions you would
like clarification on!